Chapter 17

Practice Exam 2: Answers and Explanations

If this is the second practice exam you’ve taken, we hope you’ve improved your scores and are feeling more confident about taking the Praxis Core. Use the answers and explanations in this chapter to see how well you performed and to understand where you might have gone wrong on the answers you missed. Remember, the practice exam can help you determine where you need to focus your studies in preparation for the real Praxis Core. If you want to score your test quickly, flip to the end of the chapter, where the “Answer Key” gives only the letters of the correct answers.

tip If you want even more preparation for taking the Praxis, head to learn.dummies.com. There you can take three more practice tests, review the answers and explanations, and get a personalized summary of your performance.

Part 1: Reading

  1. E. change his legacy.

    The passage explains that Nobel was dismayed by seeing himself referred to as “the merchant of death” in a newspaper. He didn’t want to be remembered that way, so he endowed the Nobel Prizes so that he would be remembered for something else. In other words, he wanted to change his legacy.

    The right answer is not Choice (A) because the passage never indicates that Nobel made any money from the Nobel Prizes; on the contrary, he devoted the fortune he had made through other means to their establishment.

    The right answer is not Choice (B) because there’s no indication that Nobel thought he would actually live longer as a result of endowing the Nobel Prizes. The idea was to be remembered differently, not to live longer.

    The right answer is not Choice (C) because, though Nobel did want the prizes to draw attention away from other things he had done, he wasn’t specifically trying to cover those things up. People would still know that he had invented explosives; he would just be more famous for something else.

    The right answer is not Choice (D) because the passage doesn’t indicate that Nobel’s motivation was to be famous forever as a result of endowing the prizes. He was already famous, but he wanted to be remembered for doing a good thing instead of causing destruction.

  2. C. report on new and exciting discoveries about killer whales.

    The passage reports on recent discoveries about killer whales. As is often the case with such questions, the broadest answer is the correct one.

    The right answer is not Choice (A) because, although the passage mentions that there are killer whale subspecies, it doesn’t explain how they diverged or even focus entirely on the subspecies topic.

    The right answer is not Choice (B) because the passage never implies any sort of disagreement between scientists and the general public.

    The right answer is not Choice (D) because even though the passage closes by mentioning that most scientists now use the term orca, the passage as a whole is not an argument about proper terminology.

    The right answer is not Choice (E) because the passage is not particularly humorous and, although it’s not written for an audience of experts, there’s no indication that it’s aimed specifically at children.

  3. C. evidence that nobody actually lived there.

    The middle of the passage not only mentions the lack of evidence that Göbekli Tepe was residential but also establishes that the site dates from before humans established permanent residences at all. Therefore, its purpose must have been ceremonial.

    The right answer is not Choice (A) because there’s no reason to assume that an archaeological site was religious just because it’s large. Religious structures are often large, but so are other types of structures.

    The right answer is not Choice (B) because, while the passage explains that Göbekli Tepe had elaborate artwork, that fact in and of itself is not the primary evidence that it was a religious site.

    The right answer is not Choice (D) because, though the passage does briefly compare Göbekli Tepe to Stonehenge, it doesn’t imply a “close resemblance.”

    The right answer is not Choice (E) because the passage doesn’t say that Göbekli Tepe played a role in the invention of agriculture; it mentions that people hadn’t developed agriculture when Göbekli Tepe was built, as support for the conclusion that the structure could not have been a permanent residence.

  4. D. inform readers about Hoffman and the controversy surrounding her work.

    The best answer here is Choice (D) because the passage does inform readers about Hoffman and about the controversy surrounding her work. Other answers may be close, but Choice (D) is the most accurate description of the overall passage content.

    The right answer is not Choice (A) because, while the passage does include details about Hoffman’s life and work, it doesn’t include enough personal detail to be called “a personal biography.”

    The right answer is not Choice (B) because the passage doesn’t argue a specific viewpoint on the Field Museum’s decision.

    The right answer is not Choice (C) because the passage as a whole is about Hoffman and her work. It mentions that some people were offended by Hoffman’s work and establishes what their reasoning was, but that doesn’t serve as the basis of the passage.

    The right answer is not Choice (E) because, although the passage establishes that there haven’t been many famous female sculptors, that point isn’t the main topic.

  5. A. trailblazing.

    Pioneering is a synonym for trailblazing — breaking new ground in a certain area (in Hoffman’s case, being among the first famous female sculptors).

    None of the other choices is correct because they aren’t what pioneering means in this context. Many of them may be words that accurately describe Hoffman or her work, but they don’t fit what the question is asking for.

  6. B. multicultural sensitivity with celebration of the achievements of women.

    The thorny paradox raised by the passage is that, although Hoffman’s work was potentially offensive from a multiculturalist perspective, the same sensitivity that demands we address this also encourages us to celebrate the work of important women in the arts.

    The right answer is not Choice (A) because how would we even know what the cultural values of the future are? The passage does not make any predictions.

    The right answer is not Choice (C) because the passage doesn’t set up any kind of paradox that pits science against art; rather, it pits the duty to honor women artists against the duty to respect the views of people who might object to her work.

    The right answer is not Choice (D) because the passage doesn’t pit intentionalist criticism (that is, approaching works of art from the perspective of the artist’s original intentions) against interpretive criticism (prioritizing that art’s effects on the audience). Even if you didn’t recognize these terms, it should still be possible to eliminate this answer choice. It was there to try to trick people who automatically gravitate toward the most confusing answer choice.

    The right answer is not Choice (E) because the passage doesn’t really get into what the mission of a museum is. You can reasonably infer that the Field Museum decided to hide the sculptures because of public opinion, but the passage is about Hoffman’s work itself, not the museum and its mission.

  7. A. It examines a familiar concept from an unorthodox viewpoint.

    The familiar concept is the Renaissance, and the unorthodox viewpoint is the notion that it was caused by the preceding Black Plague.

    The right answer is not Choice (B) because the passage doesn’t address or imply any conspiracy theory.

    The right answer is not Choice (C) because the passage never brings up a two-sided controversy.

    The right answer is not Choice (D) because the passage discusses only one artistic period — the Renaissance — not two.

    The right answer is not Choice (E) because, though the passage mentions how the Black Plague came to Europe, it doesn’t scientifically analyze the origins of the epidemic.

  8. A. “artistic brilliance”

    The preoccupation with the “human experience” mentioned in the second paragraph led to the “artistic brilliance” mentioned in the first. The Renaissance is primarily associated with the arts, and the other part of the sentence in the second paragraph clearly alludes to the “scientific advancement” previously mentioned, so the rest of the sentence is presumably about the arts.

    The right answer is not Choice (B) because the first half of the sentence (“desperation to stop the devastation had gotten people thinking …”), not the second, is what alludes to the “scientific advancement” mentioned in the first paragraph.

    The right answer is not Choice (C) because the first paragraph mentions “violent political oppression” as a characteristic of the Dark Ages, not of the Renaissance.

    The right answer is not Choice (D) because the “European golden age” indicates the Renaissance as a whole, not specifically the artistic side of it.

    The right answer is not Choice (E) because “Europe’s worst nightmare” refers to the Black Plague itself, not to the effect it had on people’s relationship to art.

  9. E. coherent

    The feudal system collapsed after the Plague, so it was no longer feasible or coherent (it couldn’t “cohere” or “hold together”).

    None of the other choices is correct because none gives a correct definition of feasible.

  10. C. Laborers became so scarce that they were in a better bargaining position.

    The passage explains that the peasant population was so devastated by the Plague that laborers were suddenly in a better position simply because there were fewer of them.

    The right answer is not Choice (A) because, although advances in ethical philosophy were made during the Renaissance, those advances aren’t why feudalism ended. The collapse of feudalism was an immediate effect of the Plague, and the advancements of the Renaissance were an eventual effect.

    The right answer is not Choice (B) because scientific discoveries aren’t what gave the emergent middle class a wider range of job options in the early Renaissance. The opportunities occurred because so many commoners died during the Plague.

    The right answer is not Choice (D) because trade between Asia and the Mediterranean is mentioned in the passage as contributing to the outbreak of the Plague in Europe, not as having led to “increased mechanization.”

    The right answer is not Choice (E) because the passage doesn’t mention feudal lords fleeing their castles and abandoning their lands. They stayed in their castles, but there weren’t enough peasants left to farm their lands anymore.

  11. B. The Renaissance was the inevitable result of the vastly reduced population.

    The meaning of the given phrase, and the point of the passage as a whole, is that the Renaissance came about more or less because Europe’s population was thinned out by the Plague.

    The right answer is not Choice (A) because the passage never implies that the Renaissance didn’t happen. The passage just makes an argument about what caused it.

    The right answer is not Choice (C) because the passage says that the Black Plague, not the Renaissance, started in central Asia.

    The right answer is not Choice (D) because the passage never implies that the Renaissance actually happened at a different time from when it reportedly did; it just makes an interesting argument about its causes.

    The right answer is not Choice (E) because, though the passage does indeed offer a dark explanation for the causes of the Renaissance, it doesn’t suggest that the term “Renaissance” itself is a euphemistic (whitewashed) expression signifying something else.

  12. B. correct a widespread misconception.

    The author starts off by calling a term “misunderstood” and then goes on to explain what it means “in actuality.” The goal is clearly to correct a widespread misconception.

    The right answer is not Choice (A) because the passage is not espousing (arguing for) a particular philosophy; it’s correcting the erroneous usage of a particular term. Simply pointing out a correct definition is not the same as making an argument for a certain viewpoint.

    The right answer is not Choice (C) because the author is not taking sides in a philosophical debate; no “debate” exists here. A popular misunderstanding is not a philosophical debate.

    The right answer is not Choice (D) because the author is not suggesting a new definition, just explaining what the original definition actually is.

    The right answer is not Choice (E) because the correct usage of a term is not an “unpopular viewpoint.” Even if most people misunderstand the term, the simple correction of a definition is not a debate.

  13. C. philosophers use to describe others’ work rather than their own.

    The author explains that actual philosophers always use the term nihilism pejoratively, as an insult for others’ work, not as a description of their own.

    The right answer is not Choice (A) because the author explains that philosophers do in fact use the term nihilism; they just do so as an insult.

    The right answer is not Choice (B) because among actual philosophers, the term nihilism always means the same thing. Non-philosophers are the people who use it incorrectly.

    The right answer is not Choice (D) because the passage never implies that philosophers used the term nihilism during one particular era of philosophy only.

    The right answer is not Choice (E) because the passage establishes that young people accidentally misuse the term nihilism, which directly refutes the idea that they misuse it deliberately.

  14. D. dispassionate.

    The author is simply explaining something in an unemotional fashion. The term for that sort of tone is dispassionate.

    The right answer is not Choice (A) because the author is not using irony or cracking jokes at anyone’s expense, so calling the tone sardonic isn’t accurate.

    The right answer is not Choice (B) because the author is not talking down to anyone, so the tone isn’t condescending.

    The right answer is not Choice (C) because the author doesn’t seem to be particularly merry or gleeful, so the tone doesn’t qualify as jovial.

    The right answer is not Choice (E) because the author doesn’t seem to be frightened, wary, or paranoid about anything, so the tone isn’t apprehensive.

  15. E. jargon.

    The word jargon refers to any language that would be used by the members of a certain group among themselves but not recognized by outsiders without an explanation, so to call these terms jargon is accurate.

    The right answer is not Choice (A) because the terms aren’t being used ironically; the definitions given are what they actually mean to people in the ASMR community.

    The right answer is not Choice (B) because the words in italics aren’t being emphasized more than others; they have special formatting because they are unfamiliar.

    The right answer is not Choice (C) because even though some of the terms are in fact proper nouns, the italics don’t identify that characteristic. The term YouTube is a proper noun, and it isn’t italicized in the passage.

    The right answer is not Choice (D) because the terms aren’t being misused. They may be unfamiliar to most people, but they are being used to mean what they actually mean.

  16. C. use a specific animal as an example to explain a particular term.

    The passage begins by talking about the dunnart, but it then becomes clear that it’s using the dunnart just as a springboard to talk about the larger concept of convergent evolution. So this reading is one of those “specific example to general principle” passages that are such a common presence on tests like the Praxis.

    The right answer is not Choice (A) because, even though the passage briefly compares a particular marsupial to a particular non-marsupial, the purpose of the passage as a whole is not to distinguish between marsupials and non-marsupials. (If it were, the passage would have explained the difference.)

    The right answer is not Choice (B) because, although the passage does specify that dunnarts aren’t mice, the passage as a whole is about convergent evolution, not specifically about mice.

    The right answer is not Choice (D) because, while the passage does correct people, it would be a bit much to say it excoriates them. To excoriate is to severely and angrily criticize, so this would be a wild exaggeration of the passage’s tone.

    The right answer is not Choice (E) because the passage doesn’t say or imply that the dunnart itself is key to evolutionary theory; it’s an example of convergent evolution, which is an important concept, but lots of animals are examples of this process.

  17. C. It does not in fact provide a legal excuse for violence.

    This answer choice is essentially a paraphrase of the final sentence of the passage, so it’s definitely a point that the passage makes.

    The right answer is not Choice (A) because the passage implies that lots of people don’t know what the Chaplinsky decision means, not that no one does.

    The right answer is not Choice (B) because the passage never mentions anything about Chaplinsky being affected in any way by subsequent Supreme Court decisions.

    The right answer is not Choice (D) because, although the defendant claimed the law was unconstitutional, the passage never takes a stance on this issue.

    The right answer is not Choice (E) because one of the answer choices was in fact correct.

  18. C. explain that a very famous character was heavily based on a lesser-known one.

    The passage does in fact explain that a famous fictional detective (Sherlock Holmes) was based on a lesser-known one (Auguste Dupin); that’s all the passage does, so Choice (C) is the correct answer. In general, answer choices that use broad terms such as explain are more likely to be correct than answer choices that use more specific words such as argue or accuse.

    The right answer is not Choice (A) because the passage specifies that Arthur Conan Doyle “admitted the obvious influence and gave Poe credit,” so plagiarism isn’t an issue.

    The right answer is not Choice (B) because the passage doesn’t get into whose stories were better from a critical perspective.

    The right answer is not Choice (D) because the traditional explanation has always acknowledged, as did Conan Doyle himself, that Poe invented the detective story. Poe’s stories aren’t as broadly famous as Conan Doyle’s, but critics have always known that Poe’s came first.

    The right answer is not Choice (E) because the passage doesn’t address any confusion about Holmes and Dupin. It says that most people have heard of Holmes but not Dupin, not that people confuse the two.

  19. A. conjecture.

    The fact that the sentence begins with “perhaps” is a clue that the final sentence of the passage is conjecture — a guess or hypothesis.

    The right answer is not Choice (B) because the final sentence is not a clue to what comes later, so it’s not foreshadowing.

    The right answer is not Choice (C) because synecdoche means “using a part to symbolize a whole,” as in when you refer to your car as your “wheels.” This figure of speech doesn’t occur in the final sentence. (Don’t be fooled into picking an answer just because it’s a fancy word; these options are often tricks.)

    The right answer is not Choice (D) because an allusion is a reference, and the final sentence doesn’t contain a reference to anything.

    The right answer is not Choice (E) because poetic license refers to an author describing something in an inaccurate or ungrammatical way for the sake of aesthetics, and the final sentence doesn’t do this.

  20. B. gauche.

    “Socially oblivious” in this context means “rude” or “unmannerly,” and gauche is a fancy synonym for such terms.

    None of the other choices is correct because none presents a close synonym of “socially oblivious.”

  21. A. intellectual disdain.

    The author spends the rest of the passage explaining why the “rule of thumb” alluded to at the outset is a stupid idea, so you can fairly say that the author has “intellectual disdain” for it. (Disdain means “contempt” or “revulsion.”)

    The right answer is not Choice (B) because the author may be cynical but doesn’t appear to be shocked by anything.

    The right answer is not Choice (C) because the author doesn’t regret anything or seem to be melancholy.

    The right answer is not Choice (D) because the author is discussing a problem that exists, not delusionally imagining one.

    The right answer is not Choice (E) because, while the author is dismissing the idea that he criticizes, he is not being dismissive. He wants to combat the problem, not ignore it.

  22. E. self-contained.

    Discrete means “separate,” so the best answer is Choice (E), self-contained. (Remember, don’t confuse discrete with discreet, meaning “secret.”)

    None of the other answers is correct because none is the definition of discrete.

  23. C. naïve administrators.

    The second sentence of the passage lays the blame on administrators, and the passage in general explains why their policy is naïve.

    None of the other choices is correct because the second sentence of the passage establishes that the author blames administrators. Nowhere does the passage indicate that the author blames teachers, students, scientists, or parents. It never mentions scientists or parents at all.

  24. B. draw the line at respecting opinions that are demonstrably untrue.

    The last sentence of the first paragraph and the first two sentences of the second are the places where the passage most clearly states that the author’s complaint is with professors being obliged to respect ideas that aren’t true.

    The right answer is not Choice (A) because the author never says or implies that professors should deliberately offend students. The point is that offense is sometimes unavoidable in education, not that it’s desirable.

    The right answer is not Choice (C) because the author’s point is that facts need to be respected over opinions; the passage never suggests that professors should simply teach their own personal opinions.

    The right answer is not Choice (D) because this statement is nearly the opposite of the author’s point; the passage argues that professors should correct students’ opinions and not merely their mechanics.

    The right answer is not Choice (E) because the author never implies that professors should go out of their way to flunk students who appear unknowledgeable. The passage simply says that correcting students’ general knowledge should be part of a professor’s job.

  25. C. whether to retreat from the world or try to improve it.

    Choice (C) is a clear paraphrase of the line from the passage that says “should a good person ‘turn the other cheek’ in the face of evil … or should he attempt to make the world a better place by actively combating the wicked?”

    The right answer is not Choice (A) because, early on, the author flatly states that the famous soliloquy is “not about whether to commit suicide.”

    The right answer is not Choice (B) because, although the author explains that the paradox Hamlet confronts involves whether to take a certain piece of scriptural advice, the passage never indicates that the question is about God’s existence itself.

    The right answer is not Choice (D) because the author explains that the speech is about whether to fight evil at all or to passively ignore it, not about the methods one may use to fight it.

    The right answer is not Choice (E) because the passage doesn’t imply that “To be or not to be” is about something incomprehensible; the author explains one possible meaning.

  26. E. unacceptable.

    The author states that “Hamlet already flatly ruled out suicide on ethical grounds” — that is, he decided suicide is unacceptable.

    The right answer is not Choice (A) because the author states that Hamlet “flatly ruled out suicide on ethical grounds,” which would indicate that suicide itself is unethical. And there’s no indication Hamlet considers suicide the lesser of two evils.

    The right answer is not Choice (B) because the “eternal paradox of morality” referred to in the passage is about whether to be active or passive in the face of evil, not about suicide.

    The right answer is not Choice (C) because, although the idea that suicide is “attractive but frightening” is the common interpretation of the speech, the passage establishes early on that the author believes that interpretation to be wrong.

    The right answer is not Choice (D) because nothing in the passage points to this answer.

  27. D. correct a misreading.

    In the third sentence, the author states that “virtually everyone … is dead wrong about what [the speech] means” and then goes on to explain why. This establishes that the author’s goal is to “correct a misreading.”

    The right answer is not Choice (A) because the author explains that Hamlet is wondering about whether to take an ethical stand; it’s not the author’s own goal in writing the passage.

    The right answer is not Choice (B) because the author is trying to explain a complex piece of literature, not to simplify a complex issue. If anything, the author’s reading of the speech is more complex than the common one.

    The right answer is not Choice (C) because the author never warns about anything that might happen in the future.

    The right answer is not Choice (E) because the author explains that other people often make a mistake in explaining the “to be or not to be” speech; the passage never indicates the author made a mistake or apologizes for having made one.

  28. A. Why should great apes have rights similar to human rights?

    The final sentence, which begins with “The reasoning goes that …”, is where the passage does indeed answer the question of why great apes should have rights similar to human rights.

    The right answer is not Choice (B) because the passage never fully explains the difference between a great ape and a monkey. Monkeys are never mentioned.

    The right answer is not Choice (C) because, although the passage mentions that the founders of the Great Ape Project are Peter Singer and Paola Cavalieri, it never says that they’re its most famous supporters. There may be more famous supporters whom the passage simply never mentions.

    The right answer is not Choice (D) because the passage mentions that great apes are both exhibited in zoos and used in scientific research, but it never says which purpose more great apes are used for.

    The right answer is not Choice (E) because, although the passage mentions that great apes are intelligent, it never explains how researchers know this or how apes’ intelligence is measured.

  29. D. They do not make use of underground burrows.

    The second sentence of the passage states that hares live in nests and that rabbits are the ones who live in burrows.

    The right answer is not Choice (A) because that passage says that the hare is the only mammal with a jointed skull, not that it’s the only animal with one.

    The right answer is not Choice (B), (C), or (E) because all these statements are true of rabbits, not hares.

  30. C. Matter can be sent into the past, but not living beings.

    The passage objects to the idea of time-travel to the past by explaining that a human traveling to the past may cause a paradox. The idea that inanimate matter could be sent to the past, but not living beings, would get around the paradox, and thus resolve the objection.

    The right answer is not Choice (A) because, though physicists do indeed believe exceeding the speed of light to be impossible, the passage doesn’t raise any objection to a time-travel theory based on this issue. So a discovery that surpassing the speed of light is somehow possible would not resolve an objection made in the passage.

    The right answer is not Choice (B) because physicists already believe that exceeding the speed of light is impossible, so a confirmation of this fact wouldn’t explain anything.

    The right answer is not Choice (D) because the passage never mentions other dimensions, so the discovery would be irrelevant to the claims of the passage.

    The right answer is not Choice (E) because the passage doesn’t present any definitions of time, so the discovery would be irrelevant to the claims of the passage.

  31. B. exceptions and qualifications.

    The expression fine print (like the words in small font at the bottom of a contract, where most of the catches are hidden) usually means something like “exceptions and qualifications,” and that’s what it means in this context as well; time travel is possible, but not in the way you probably think.

    None of the other choices is correct because none is the definition of fine print as it’s used in the passage.

  32. D. It’s a natural consequence of the relationship between speed and time.

    The passage closes by saying that a time machine could be anything that goes fast enough — that is, that time travel is a natural consequence of the relationship between speed and time.

    The right answer is not Choice (A) because the passage never says that humans have already achieved time travel.

    The right answer is not Choice (B) because the passage never says that time travel to the future would resolve its own paradoxes, just that it wouldn’t create any.

    The right answer is not Choice (C) because, although the passage does explain that time travel to the future is to some extent a matter of perspective, that statement doesn’t make time travel the same as an illusion. (It’s actually happening, but only happening relative to something else.)

    The right answer is not Choice (E) because, though the simple passage of time does technically constitute time travel to the future (in other words, you’re traveling to the future right now because time is passing), that topic isn’t addressed in the passage.

  33. D. Passage 1 explains the origin of a theory that Passage 2 then modifies.

    Passage 1 explains where the asteroid theory came from, and Passage 2 then adds to the asteroid theory, rather than presenting a competing theory.

    The right answer is not Choice (A) because the theories presented in the two passages aren’t in fact mutually exclusive (a term meaning they can’t be true at the same time). Passage 2 admits that the asteroid impact happened; it just argues that other things happened too.

    The right answer is not Choice (B) because, though Passage 2 admits that the asteroid impact happened, that doesn’t mean that the two passages are exploring the same theory from different angles. Passage 1 concerns the theory that the dinosaurs were killed by just an asteroid, and Passage 2 concerns the theory that they were killed by an asteroid and other events as well.

    The right answer is not Choice (C) because the theory in Passage 2 doesn’t seek to replace the theory in Passage 1, just to modify it. Besides, to say that it has replaced the theory in Passage 1 would be incorrect because the readings establish that the theory from Passage 1 is still more popular.

    The right answer is not Choice (E) because Passage 1 and Passage 2 present two (marginally) different theories, not the data and the conclusions for one single theory.

  34. A. ignoring.

    When Passage 2 says that attributing the extinction solely to an asteroid would involve “writing off” the Deccan Traps, the best paraphrase is to say it would involve ignoring the Deccan Traps role in the extinction.

    The right answer is not Choice (B) because the objection made by Passage 2 is that the asteroid theory involves believing that the Deccan Traps played no role in the extinction — that is, that the Deccan Traps are being ignored, not misrepresented.

    The right answer is not Choice (C) because Passage 2 doesn’t indicate that the theory from Passage 1 is obscuring (covering up or hiding) the role played by the Deccan Traps. It means that the theory from Passage 1 simply doesn’t account for the Deccan Traps having played a role.

    The right answer is not Choice (D) because the complaint of Passage 2 is that the Deccan Traps aren’t being credited with enough of a role in the extinction, which is the opposite of saying that their role is being exaggerated.

    The right answer is not Choice (E) because Passage 2 isn’t accusing anyone of plagiarizing anything; it’s saying that important data is being ignored, not stolen.

  35. C. One author believes it killed the dinosaurs, and the other believes it was one of multiple causes.

    Passage 1 concerns the theory that the asteroid killed the dinosaurs, and Passage 2 concerns the theory that at least one other cause may have been involved. The place to confirm this answer is the final sentence of Passage 2 (“No one is denying …”).

    The right answer is not Choice (A) because saying that the author of Passage 2 doesn’t believe an asteroid killed the dinosaurs is going too far.

    The right answer is not Choice (B) because both authors agree about the timing of the asteroid impact.

    The right answer is not Choice (D) because the author of Passage 1 characterizes the asteroid as being heavily composed of iridium, and the author of Passage 2 never challenges this description.

    The right answer is not Choice (E) because the author of Passage 1 never appears to doubt that the asteroid theory is correct.

  36. E. Passage 1 explains the history of its theory, whereas Passage 2 does not.

    Much of Passage 1 consists of an explanation of when and how the asteroid theory was developed and supported. Passage 2 never gets into the history of the idea that the Deccan Traps played a role.

    The right answer is not Choice (A) because the two passages aren’t each based on a different scientific discipline. They both involve both chemistry and physics. (Also, keep in mind that an answer choice that would require you to have outside knowledge of the topic is extremely unlikely to be correct; this is a reading test, not a science test.)

    The right answer is not Choice (B) because, although the passages establish that the asteroid hit what is now North America and the Deccan Traps formed in what is now Asia, neither passage says that either of these locations is the only place dinosaurs lived.

    The right answer is not Choice (C) because the two passages are very similar in tone and reading level, so there’s no reason to assume that they were composed with different audiences in mind.

    The right answer is not Choice (D) because there’s no indication that the author of Passage 2 formulated the Deccan Traps theory, so it’s not an original (to the author) theory.

  37. B. limited.

    The theory from Passage 1 suggests one cause, and the theory from Passage 2 puts forth multiple causes that include the cause from Passage 1. Therefore, Passage 2 sees Passage 1 as limited — that is, true but incomplete.

    The right answer is not Choice (A) because Passage 2 admits that the asteroid impact happened and played a role.

    The right answer is not Choice (C) because Passage 2 never implies that the theory in Passage 1 is old and that its own idea is newer.

    The right answer is not Choice (D) because Passage 2 never implies that the theory in Passage 1 is contradicting itself somehow.

    The right answer is not Choice (E) because there’s no indication that the author of Passage 2 sees the theory from Passage 1 as elitist (somehow snobbish or exclusionary).

  38. D. the causes of an agreed-upon event.

    The “agreed-upon event” is the extinction of the dinosaurs, and both passages discuss its possible causes.

    The right answer is not Choice (A) because the two authors agree about when dinosaurs lived, which you can tell by the fact that they’re both discussing the extinction as having happened at the same time.

    The right answer is not Choice (B) because neither author ever says anything about some dinosaurs disappearing at different times from other dinosaurs.

    The right answer is not Choice (C) because the author of Passage 2 concedes that the asteroid struck earth, and that it did so at the time that the theory in Passage 1 claims.

    The right answer is not Choice (E) because neither author ever addresses the question of why one theory or another is popular, or even talks much about which theory is more popular.

  39. C. the Medieval Period and the Renaissance.

    The Middle Ages and the Medieval period are two terms for the same time in history, and the Renaissance came after it. The correction the passage makes is that suits of armor are actually from the Renaissance, not from the Medieval period.

    The right answer is not Choice (A) because, although the passage begins by mentioning that most children would be mistaken about when suits of armor were actually worn, it also says that most adults would, too.

    The right answer is not Choice (B) because the passage doesn’t distinguish between armor and regular clothing; it distinguishes between the armor of two time periods.

    The right answer is not Choice (D) because, although the passage establishes that European warriors were the ones who wore suits of armor, the passage doesn’t involve comparing them to warriors from other places.

    The right answer is not Choice (E) because the passage actually compares wealthy and less-wealthy warriors, saying essentially that all Medieval warriors would have worn plate-and-mail suits.

  40. C. civilization hasn’t existed long enough to substantially alter human instinct.

    The point of the passage as a whole, as stated most clearly in the last two sentences, is that human instinct and genetics were formed for millions of years, and that civilization hasn’t existed for nearly long enough to have substantially altered these things.

    The right answer is not Choice (A) because the passage never suggests that common people are broadly wrong about how long civilization has been around; rather, the point is that most people don’t know how long humans were evolving before that time.

    The right answer is not Choice (B) because the point of the passage is that the development of civilization (government, agriculture, and so on) didn’t substantially affect human genetics, simply because it happened too recently to have done so yet.

    The right answer is not Choice (D) because the passage certainly doesn’t say that human evolution stopped when civilization began, just that the previous evolutionary trajectory hasn’t yet been significantly altered by civilization.

    The right answer is not Choice (E) because the passage never implies that experts disagree about any of this information; it says that laypeople misunderstand it.

  41. B. apparently straightforward questions can be muddled by vague definitions.

    Ultimately, the point of the passage is to call attention to the fact that the apparently straightforward question “What is the southernmost city in the world?” is not so easy to answer because it depends on what counts as a city.

    The right answer is not Choice (A) because the point of the passage is not that geography is complex. The problem at hand is how many people a community needs in order to count as a city, which is not a geographic question.

    The right answer is not Choice (C) because the passage never implies that the argument is silly. The author appears to think that the question is valid and interesting, though apparently unsolvable.

    The right answer is not Choice (D) or Choice (E) because the passage never takes one country’s side over the other’s; it just explains the nature of the dispute.

  42. B. The first paragraph mainly presents details of William Tyndale’s life, and the second deals more closely with his influence.

    The first paragraph is mainly about Tyndale’s life, and the second paragraph is mainly about his influence, so Choice (B) is clearly the best description.

    The right answer is not Choice (A) because both paragraphs are mainly about Tyndale, not just the second one.

    The right answer is not Choice (C) because the first paragraph is about Tyndale’s life, not the influence of his Bible.

    The right answer is not Choice (D) because the first paragraph is much more about Tyndale and only briefly mentions Shakespeare. Though the second paragraph does allude to Tyndale’s influence on Shakespeare, that’s not the main point of the paragraph.

    The right answer is not Choice (E) because the first paragraph is mainly about Tyndale, not other Bible translations done before the invention of the printing press. (It only briefly alludes to Wycliffe’s translation.) Furthermore, the second paragraph is specifically about Tyndale’s Bible, not printed Bible translations in general.

  43. A. use him as the standard by which influence on the English language is measured.

    Both allusions to Shakespeare invoke him as the greatest influence on the English language, in order to praise Tyndale by establishing his influence on both the language and on Shakespeare himself.

    The right answer is not Choice (B) because only the second paragraph discusses Tyndale’s influence on Shakespeare himself, and even then, it never says that Tyndale was the greatest influence on Shakespeare, just a major influence (don’t pick a choice that goes further than you need it to).

    The right answer is not Choice (C) because the passage never directly compares Tyndale’s language to Shakespeare’s.

    The right answer is not Choice (D) because the passage never implies that people confuse figures of speech coined by Tyndale with those coined by Shakespeare.

    The right answer is not Choice (E) because, although the passage certainly elaborates on the political constraints on Tyndale’s work, it never mentions anything about political constraints on Shakespeare’s.

  44. E. The character of modern literary English was largely established in the 16th century.

    The passage cites Shakespeare and Tyndale as the two greatest influences on the form of literary English, and it establishes that they both did their work in the 16th century.

    The right answer is not Choice (A) because the passage says that Shakespeare was influenced by Tyndale’s use of language in his translation of religious texts, not necessarily by religion itself.

    The right answer is not Choice (B) because the passage never addresses the question of political differences among various English translations of the Bible.

    The right answer is not Choice (C) because nothing in the passage implies that Tyndale’s Bible caused Henry VIII’s break with Rome. The passage merely explains that the Roman Catholic Church was against English translations of the Bible, and that such translations (like Tyndale’s, even though he was already dead) became acceptable in England after the king broke with the Catholic Church.

    The right answer is not Choice (D) because the passage never says that translations done by individuals are generally superior to those done by committees.

  45. C. who attempted a similar project to Tyndale’s but was thwarted.

    The lone mention of Wycliffe explains that, like Tyndale, he tried to translate the Bible into English, but was thwarted because his handwritten translations made it much easier for the state to seize and destroy most of the copies.

    The right answer is not Choice (A) because the passage never says that Tyndale was directly inspired by Wycliffe, just that they both tried to do the same thing.

    The right answer is not Choice (B) because the passage never says Wycliffe was as talented as Tyndale or mentions anything about Wycliffe’s politics.

    The right answer is not Choice (D) because the passage doesn’t say that Wycliffe’s influence on English is harder to measure than Tyndale’s; on the contrary, you can safely assume that Wycliffe’s influence was much smaller because his Bibles were destroyed.

    The right answer is not Choice (E) because the passage never says Wycliffe was more religious than Tyndale.

  46. D. knack for.

    To have an “ear for” something, in music or in writing, is to be finely attuned to its capacity for aesthetic beauty — that is, to have a knack for it.

    None of the other choices is correct because, in this context, no other answer is an appropriate synonym for the phrase “ear for.”

  47. 2010.

    All you need to do is look for the year above which the bottom and the middle lines are farthest apart. That’s 2010, when there were apparently about 6 wolves in the Wenaha pack and 15 in the Imnaha pack.

  48. C. 2011 and D. 2012.

    In the years 2009 and 2010, the numbers indicated by the bottom and middle lines seem to add up to the number indicated by the top line — that is, the Wenaha and Imnaha wolf packs’ populations added up to the total wolf population of the state. Thus, the right answer is not Choice (A) or Choice (B). For the years 2011 and 2012 (Choices [C] and [D]), however, the number indicated by the top line is greater than the sum of the numbers indicated by the bottom and middle lines, indicating that wolves that belonged to packs other than the Wenaha and Imnaha were in the state.

  49. B. 2010.

    This question is a tricky one; you have to look at the closeness of the middle and top lines, but you also have to consider percentages. In 2009, there looked to be about 15 wolves total (top line) and 10 in the Imnaha pack, so that means that 2/3 of the wolves in the state, or about 67 percent, were from the Imnaha pack. In 2010, the top and middle lines are also close together, but there look to be roughly 20 wolves total and 15 in the Imnaha pack, which means that 3/4 of the wolves in the state, or about 75 percent, were from the Imnaha pack. The answer is 2010, because 75 percent is more than 67 percent.

    The right answer is not Choice (A), (C), or (D) because the percentage of total wolves that were from the Imnaha pack in those years is not the highest of the yearly percentages.

    The right answer is not Choice (E) because the graph does in fact reveal the requested information.

  50. A. Both psychopathy and sociopathy are observable via brain autopsy.

    The middle of the passage states that, concerning both psychopathy and sociopathy, “there is agreement that the conditions involve observable differences in brain structure.”

    The right answer is not Choice (B) because the passage states that only some professionals think that both psychopathy and sociopathy are synonymous with antisocial personality disorder.

    The right answer is not Choice (C) because the passage states that only some professionals use the terms psychopath and sociopath interchangeably.

    The right answer is not Choice (D) because the passage establishes (at the end) that, according to how an increasing number of professionals use the term, only psychopaths genuinely have no sense of right and wrong.

    The right answer is not Choice (E) because the passage states that psychopathy and sociopathy are difficult, but not impossible, to detect via psychological examination.

  51. E. clear up confusion on the parts of general readers.

    The passage aims merely to distinguish the ways that psychological professionals currently use the terms psychopath and sociopath.

    The right answer is not Choice (A) because the passage is explanatory and informative, not critical — it never says that anyone is wrong.

    The right answer is not Choice (B) because the passage never expresses a personal opinion; the author only explains what other people think.

    The right answer is not Choice (C) because the passage never mentions any particular person; it’s just about the terms psychopath and sociopath in general.

    The right answer is not Choice (D) because, though the passage mentions that psychopaths often try to evade detection, the goal is not to warn professionals about this fact; the passage establishes that professionals already know it.

  52. C. logically expectable.

    When the passage describes the average person’s confusion as “forgivable,” it does so as a way of introducing the idea that many professionals use the terms psychopath and sociopath in different ways. The idea is that the layperson’s confusion is unavoidable and understandable, so the most correct answer is Choice (C).

    The right answer is not Choice (A) because, although the term forgivable sometimes means “morally excusable,” the passage doesn’t regard the average person’s confusion as a moral issue, so Choice (A) is not the best of the choices in context.

    The right answer is not Choice (B) because the passage doesn’t say that the average person’s confusion is confusing itself. This choice is trying to fool you by repeating words.

    The right answer is not Choice (D) because the passage never indicates that the author regards people’s confusion as either disturbing or amusing.

    The right answer is not Choice (E) because the passage never says or implies that the author considers the layperson’s confusion annoying.

  53. B. idiosyncratic.

    This is just a vocabulary question. The passage explains (at the end) that sociopaths have a sense of right and wrong that is massively out of line with most people’s sense of these things. The fancy word for “different from other people” is Choice (B), idiosyncratic.

    None of the other choices is correct because none of the other words means “different from other people.” Choice (A), ineffable, means “inexpressible.” Choice (C), inextricable, means “impossible to separate.” Choice (D), remunerative, means “yielding money or reward.” Choice (E), eleemosynary, means “related to or depending on charity.”

  54. D. multifaceted.

    The point of the passage is that Yeats’s poetry went through many different phases and had many different sides to it. The fancy word for “having many dimensions or aspects” is multifaceted, Choice (D).

    None of the other choices is correct because none represents the “having many dimensions or aspects” concept.

  55. A. the general to the specific.

    The first sentence states a widespread principle or truism, and the rest of the passage talks about a specific example of it.

    The right answer is not Choice (B) because, although the first sentence of the passage does make reference to a superstition, the point of the passage as a whole is not to distinguish superstition and fact.

    The right answer is not Choice (C) because the passage is not divided between rhetoric (attempts at eloquence or persuasion) and logic.

    The right answer is not Choice (D) because the first sentence of the passage doesn’t provide data that the rest of the passage subsequently interprets.

    The right answer is not Choice (E) because the first sentence of the passage doesn’t state a cause with the rest of the passage then delineating its effects.

  56. C. they are absent from preserved records.

    The second sentence of the passage explains that “dirty” words are usually used in speech for a long time before they’re written down in any way that survives to be studied. This gap is what makes pinning down their etymologies difficult.

    The right answer is not Choice (A) because the passage never says that linguists are reluctant to study “dirty” words; on the contrary, it says that doing so is a source of “amusement” for them.

    The right answer is not Choice (B) because, while the final sentence of the passage states that misinformation often circulates about the origins of “dirty” words, it explains that people do this mistakenly rather than deliberately.

    The right answer is not Choice (D) because the passage never says or implies that the meanings of “dirty” words change more often than those of other words do.

    The right answer is not Choice (E) because the passage explains that “dirty” words actually don’t usually originate as acronyms, although urban legends often claim that they do.

Part 2: Writing

  1. E. No error

    The sentence does not contain an error.

    The right answer is not Choice (A) because the its without the apostrophe is the possessive, so it’s correct in this context.

    The right answer is not Choice (B) because the conjunction whereas means something like “while on the other hand,” so it’s used correctly here.

    The right answer is not Choice (C) because the sentence uses the adverb simply (meaning “merely”) to clearly and correctly modify the verb use.

    The right answer is not Choice (D) because this phrasing is grammatically correct as written. You may have picked this answer thinking it was missing the word that; although the phrase “that it contains” would also be correct, the omission of that doesn’t create an error.

  2. D. by which

    A movie is based on a book, not by a book, so “on which” is the correct prepositional construction here.

    The right answer is not Choice (A) because the movies in question are both “well made” and “considered cinematic classics.” The construction is clear and correct, and no commas or other punctuation are necessary.

    The right answer is not Choice (B) because for is the correct preposition in this context.

    The right answer is not Choice (C) because saying that a movie didn’t do something “as well as did the book” (as opposed to “as well as the book did”) is perfectly correct even though it’s a bit more formal than everyday speech.

    The right answer is not Choice (E) because there is, in fact, an error in the sentence.

  3. A. people who believe

    The word who should be omitted, because the first clause needs to be independent (“people believe” is an independent subject and verb, whereas “people who believe” subordinates the verb to a pronoun, resulting in an incomplete sentence).

    The right answer is not Choice (B) because this phrase correctly uses the infinitive.

    The right answer is not Choice (C) because this is a clear and correct usage of an adverb modifying an adverb that is modifying an adjective.

    The right answer is not Choice (D) because than (not then) is the word used for comparisons.

    The right answer is not Choice (E) because there is, in fact, an error in the sentence.

  4. B. whom was

    The pronoun is the subject of a subordinate clause, so who is correct.

    The right answer is not Choice (A) because the regular past tense is fine here.

    The right answer is not Choice (C) because using a comma after a conjunction is acceptable when the sentence is interrupted by a modifying clause (such as “ironically”) at that point.

    The right answer is not Choice (D) because the participial verb form is fine here.

    The right answer is not Choice (E) because there is, in fact, an error in the sentence.

  5. C. is their

    The antecedent of the pronoun is singular (“a living organism”), so you need the singular possessive pronoun its rather than the plural their here.

    The right answer is not Choice (A) because the past tense arose is perfectly correct here.

    The right answer is not Choice (B) because the third-person singular distinguishes is correct here.

    The right answer is not Choice (D) because the subject of the verb in question is manner, so the third-person singular passes on (meaning “transmits”) is correct.

    The right answer is not Choice (E) because there is, in fact, an error in the sentence.

  6. D. myself

    The speaker is not performing a verb reflexively with herself as the object, so myself is incorrect. She should simply have said me.

    The right answer is not Choice (A) because should you is a correct construction here (to mean “If you should …”).

    The right answer is not Choice (B) because, although data is often used as a singular noun, it’s actually plural, so data represent is correct.

    The right answer is not Choice (C) because the infinitive is correct in this context.

    The right answer is not Choice (E) because there is, in fact, an error in the sentence.

  7. E. No error

    This sentence doesn’t contain any errors.

    The right answer is not Choice (A) because the its without an apostrophe is the possessive, so it’s correct in this context. (It’s with an apostrophe is the contraction for it is.)

    The right answer is not Choice (B) because although the as isn’t necessary here, including it’s not wrong. In fact, including it is more formal and correct.

    The right answer is not Choice (C) because Scots (Scottish people) is a proper noun and should be capitalized.

    The right answer is not Choice (D) because after the time (meaning after the time in which he lived) is perfectly clear and correct in this context.

  8. B. further

    Because the sentence is discussing physical distance, farther is the correct adjective.

    The right answer is not Choice (A) because the word children should indeed be possessive here (even though the museum doesn’t literally belong to actual children), and the apostrophe is in the right place.

    The right answer is not Choice (C) because than (not then) is the word used for comparisons.

    The right answer is not Choice (D) because less is correct here (as opposed to fewer, which would be used for countable quantities).

    The right answer is not Choice (E) because there is, in fact, an error in the sentence.

  9. C. in which

    You don’t hear in an album; you just hear an album, so the addition of in is unnecessary and incorrect. Plus, which is used incorrectly here; the phrase should read “that I have ever heard.”

    The right answer is not Choice (A) because very many is a perfectly clear and correct instance of an adverb modifying an adjective.

    The right answer is not Choice (B) because best-produced forms a two-word adjective here (in which the first word is not an adverb), so it should be hyphenated.

    The right answer is not Choice (D) because the present-perfect have ever heard is correct in this context.

    The right answer is not Choice (E) because there is, in fact, an error in the sentence.

  10. E. No error

    This sentence contains no error.

    The right answer is not Choice (A) because the word effect can also be a verb meaning “to bring about.”

    The right answer is not Choice (B) because then (not than) is the word used for conditional statements (“if/then”), so it’s correct here.

    The right answer is not Choice (C) because affect is a verb meaning “to change,” so it’s correct here.

    The right answer is not Choice (D) because than (not then) is the word used for comparisons, so it’s correct here.

  11. A. being

    The word being is pointlessly inserted into the appositive clause; the convention is for an appositive clause to limit itself to the definition without the gerund.

    The right answer is not Choice (B) because planet doesn’t need to be capitalized.

    The right answer is not Choice (C) because dwarfs is used correctly here as a verb meaning “to make something else seem small by comparison.”

    The right answer is not Choice (D) because it’s (with an apostrophe) is the contraction for “it is,” so it’s correct in this context.

    The right answer is not Choice (E) because there is, in fact, an error in the sentence.

  12. B. were

    The subject is treasurer, which is singular, so the verb should be was, not were.

    The right answer is not Choice (A) because “devoted to the upkeep …” is an adjectival phrase modifying society, so this construction is correct.

    The right answer is not Choice (C) because “unsure of how best to allocate” is perfectly correct even though it may be a bit more formal than everyday speech.

    The right answer is not Choice (D) because the infinitive is correct in this context.

    The right answer is not Choice (E) because there is, in fact, an error in this sentence.

  13. B. had been

    The speaker is unsure of whether Danny ever actually arrived, so the hypothetical was (“if he was here”) is correct, not the past-perfect had been.

    The right answer is not Choice (A) because you’ve been — a contraction of the present-perfect you have been — is perfectly correct in this context.

    The right answer is not Choice (C) because earlier (meaning “before now”) is correct here.

    The right answer is not Choice (D) because the regular past tense didn’t see (“did not see”) is correct here.

    The right answer is not Choice (E) because there is, in fact, an error in the sentence.

  14. D. of

    Of is the wrong preposition here; correct options include optimistic about or possibly optimistic for, but not optimistic of.

    The right answer is not Choice (A) because beginning a sentence with a subordinate because clause is fine as long as the clause is followed by a comma and an independent clause.

    The right answer is not Choice (B) because the possessive half a day’s travel (meaning “half a day’s worth of travel”) is a correct construction.

    The right answer is not Choice (C) because the clause preceding it is independent, so the clause in Choice (C) must begin with a conjunction.

    The right answer is not Choice (E) because there is, in fact, an error in the sentence.

  15. A. principle

    The words principle and principal are commonly confused. When the word means “primary or leading,” as it does in this sentence, principal is correct.

    The right answer is not Choice (B) because students give is correct subject/verb agreement.

    The right answer is not Choice (C) because the participial verb form is correct here.

    The right answer is not Choice (D) because the noun suspicion is used correctly in this context: “the reason is suspicion… .”

    The right answer is not Choice (E) because there is, in fact, an error in the sentence.

  16. D. credibly

    The proper word here is the noun credibility, not the adverb credibly.

    The right answer is not Choice (A) because the noun performing the verb is hallucinations, which is plural, so the plural verb are is correct.

    The right answer is not Choice (B) because behind is used correctly (“the explanation behind …”). You could also use of or for, but having other viable options doesn’t automatically make the given wording wrong.

    The right answer is not Choice (C) because the noun performing the verb is theory, which is singular, so the singular verb is is correct.

    The right answer is not Choice (E) because there is, in fact, an error in the sentence.

  17. D. have had for

    The sentence uses the wrong preposition here. It should say that the poems had an influence on other writers, not an influence for them.

    The right answer is not Choice (A) because the present perfect have often been dismissed is the correct tense in this context.

    The right answer is not Choice (B) because overly sentimental is a correct instance of an adverb modifying an adjective.

    The right answer is not Choice (C) because, although it’s interrupted by modifying words, the present perfect has more recently been given credit is correct.

    The right answer is not Choice (E) because there is, in fact, an error in the sentence.

  18. B. Kings and Queens

    The words kings and queens should not be capitalized here because they’re used in the abstract rather than as titles before the names of specific people.

    The right answer is not Choice (A) because “the question of which” is the correct construction here.

    The right answer is not Choice (C) because Glorious Revolution is a proper noun referring to a specific historical event, so it should be capitalized.

    The right answer is not Choice (D) because than (not then) is the word used for comparisons, so it’s correct here. As for the placement of the verb are, it’s correct either before or after “executive decisions.”

    The right answer is not Choice (E) because there is, in fact, an error in the sentence.

  19. A. they are

    As the rest of the sentence shows, the Dimetrodon is being discussed in the singular, so you need the singular it is here rather than the plural they are.

    The right answer is not Choice (B) because alongside is used correctly in this context.

    The right answer is not Choice (C) because using at all for the sake of emphasis is perfectly fine (though not necessary).

    The right answer is not Choice (D) because “rather a” is a correct construction in this context. You could also add was (“was rather a”), but omitting it is acceptable.

    The right answer is not Choice (E) because there is, in fact, an error in the sentence.

  20. D. is not doing his or her job

    The teacher in the sentence is singular (“a teacher”), so you need a singular verb (is), pronoun (his or her), and direct object (job). The other choices are incorrect because they use plural versions of one or more of these items.

  21. A. all with

    The sentence is correct the way it is.

    The right answer is not Choice (B) because the addition of the word in is incorrect; which all have would be acceptable, but not in which all have.

    The right answer is not Choice (C) because you need have in this context, not the participial form having. (Also, which is preferable to whom in this case because the subject is an animal rather than a person.)

    The right answer is not Choice (D) because it forms a comma splice.

    The right answer is not Choice (E) because it forms a comma splice.

  22. C. century, James Joyce

    This choice is a correct example of an introductory subordinate clause (although is a subordinating conjunction) followed by a comma and an independent clause.

    The right answer is not Choice (A) because the sentence doesn’t need both the subordinating conjunction although before the first clause and the coordinating conjunction but before the second clause. Using one or the other would be correct, but not both.

    The right answer is not Choice (B) because neither clause is independent.

    The right answer is not Choice (D) because a semicolon here would not separate two independent clauses (an although clause is subordinate).

    The right answer is not Choice (E) because although and however are doing the same job, so you don’t need both of them (and if only however were there, you’d need to add a semicolon or to start a new sentence).

  23. E. wit and to shock with

    This sentence presents a parallel-phrasing issue. The both is a clue that the sentence needs two verbs, both in the infinitive and not separated by a comma: the capacity both to delight … and to shock… . The only answer that accomplishes that is Choice (E).

    The right answer is not Choice (A) because the both is a clue that the two verbs (to delight and to shock) should be in the infinitive and not separated by a comma.

    The right answer is not Choice (B) because the both is a clue that the two verbs (to delight and to shock) should be in the infinitive and not separated by a comma.

    The right answer is not Choice (C) because this forms a comma splice (and a nonsensical one at that, since the first independent clause says both but only includes one verb).

    The right answer is not Choice (D) because the both is a clue that you need and rather than but.

  24. C. Having spotted the skyline in the distance, we decided to try to

    This sentence is a misplaced-modifier question. The modifier needs to attach to who or what spotted the skyline; as written, that subject is “attempt,” and clearly an attempt can’t spot something. The sentence is written in the second-person plural (we/us) point of view, so opening the independent clause with “we” makes sense.

    The right answer is not Choice (A) because the “attempt” isn’t who or what spotted the skyline, so the modifier is misplaced.

    The right answer is not Choice (B) because neither clause is independent.

    The right answer is not Choice (D) because this answer is a comma splice (because therefore is not a conjunction).

    The right answer is not Choice (E) because it’s a long participial phrase, not a complete sentence.

  25. D. for whom female fans showed their appreciation by throwing

    Choice (D) is the only answer choice in which all the prepositions and verb forms agree: Liszt was the artist for whom female fans showed their appreciation by throwing their undergarments. Remember, all the verbs in a sentence don’t always have to be in the same form.

    The right answer is not Choice (A) because it’s constructed to imply that the fans had an appreciation for the act of throwing undergarments itself rather than an appreciation for Liszt.

    The right answer is not Choice (B) because the past-progressive form were showing rather than the regular past tense showed is both awkward and unnecessary.

    The right answer is not Choice (C) because using the infinitive to show in place of the past-tense showed is nonsensical in context.

    The right answer is not Choice (E) because the fans showed their appreciation by throwing their undergarments (it was the means by which they showed appreciation), so you need by throwing rather than to throw.

  26. E. consuming all the planets of our solar system before collapsing

    The participial forms consuming and collapsing are appropriate to the afterthought clause that ends the sentence.

    The right answer is not Choice (A) because the and should not have a comma before it (even if it didn’t, the sentence as phrased would still be awkward).

    The right answer is not Choice (B) because it forms a comma splice.

    The right answer is not Choice (C) because a semicolon here would not separate two independent clauses (the then clause is not independent).

    The right answer is not Choice (D) because the afterthought clause’s structure isn’t parallel.

  27. B. Africa, is a biodiversity hotspot, with over 90 percent of its wildlife being found

    The subject of the sentence is Madagascar, and the verb is is; this choice sets it up so that the subject and verb are separated by an appositive clause, and the sentence ends with a prepositional afterthought clause.

    The right answer is not Choice (A) because it’s not a complete sentence (it has no main verb).

    The right answer is not Choice (C) because this is not a complete sentence (being can’t be the main verb).

    The right answer is not Choice (D) because it’s not a complete sentence (it has no main verb).

    The right answer is not Choice (E) because the inclusion of it gives the sentence two subjects, so it doesn’t grammatically agree.

  28. C. now; the rest of you, however, may

    This choice correctly joins two independent clauses with a semicolon (the second of which contains the adverb however, correctly set off with a pair of commas).

    The right answer is not Choice (A) because it forms a comma splice (however is not a conjunction).

    The right answer is not Choice (B) because it also forms a comma splice (however is not a conjunction).

    The right answer is not Choice (D) because, although the semicolon is correct, the however in the second independent clause would need to be set off with two commas.

    The right answer is not Choice (E) because it forms a comma splice (between now and the).

  29. A. than are those of

    The sentence is correct as it is. The construction may be a little more formal than you’re used to hearing in speech, but it isn’t wrong.

    The right answer is not Choice (B) because you’re comparing the works of van Gogh to the works of other painters, not to the painters themselves, so you need to say than are those of (meaning the works of) any other, rather than simply than any other.

    The right answer is not Choice (C) or Choice (D) because the more is a clue that you need than, rather than compared (the preposition you use after it — to or with — doesn’t matter, because compared itself is wrong).

    The right answer is not Choice (E) because you need than instead of compared, not both words.

  30. B. the truth is that firing

    This is the only choice that forms a coherent, correct sentence.

    The right answer is not Choice (A) because the construction “the truth of it is” would require a comma afterwards, in place of the omitted “that.”

    The right answer is not Choice (C) or Choice (D) because the presence of the word is later in the sentence causes those phrasings to become nonsensical. (Always read the whole sentence to determine whether a choice makes sense in context.)

    The right answer is not Choice (E) because it needs a comma after truly (and even then, it’s extremely awkward).

  31. B. A certain type of person is likely to tell you that poetry is read by hardly anyone, and that the few people who do read it comprise an insular cadre of specialists ignored by the culture at large.

    None of the choices is incorrect from a technical standpoint, but Choice (B) is the most concise and efficient of the answer choices.

    The right answer is not Choice (A) because this sentence is unnecessarily repetitive and could be more concise by eliminating “and that these people are.”

    The right answer is not Choice (C) because it’s unnecessarily repetitive and could be more concise in the “only an …” and “that is also …” portions.

    The right answer is not Choice (D) because it’s terribly awkward, particularly the phrase “hardly anyone who does not comprise.”

    The right answer is not Choice (E) because it’s unnecessarily repetitive and could be more concise in the “adding that …” and “specifying that …” portions.

  32. A. Leave it as it is.

    The sentence makes perfect sense exactly where it is and as it’s written.

    The right answer is not Choice (B) because the word “poetry” belongs in quotation marks; the sentence refers to the act of saying it. Plus, there’s no particular reason to italicize the word very.

    The right answer is not Choice (C) because beginning the sentence with “on the other hand” would make no sense; Sentence 5 doesn’t present a counterpoint to the previous sentence.

    The right answer is not Choice (D) because this sentence would be out of place as the second sentence of the next paragraph; the explanation it alludes to would have already begun.

    The right answer is not Choice (E) because the sentence doesn’t present a counterpoint to the previous sentence, so you don’t need to insert the word however.

  33. D. to the delight of some and the consternation of others

    The point of the paragraph is that slam poetry has both its supporters and its detractors, so the phrase “to the delight of some and the consternation of others” makes the most sense in context.

    The right answer is not Choice (A) because although definitions are often challenged, that isn’t the most relevant statement to be made about the specific topic under discussion here.

    The right answer is not Choice (B) because simply announcing that something is about to be explained is usually an unnecessary rhetorical move; another answer choice is clearly more necessary.

    The right answer is not Choice (C) because the paragraph as a whole doesn’t address any “varying degrees of success” on the part of slam poets. Another choice makes more sense in context.

    The right answer is not Choice (E) because the passage doesn’t refer to slam poets employing any “devious” or “unexpected” methods. The reference makes no sense in context.

  34. E. who typically stand at lecterns and read from notes or a published book during recitals

    Because the rest of the sentence describes the performance style of slam poets, the best choice for the underlined portion is the phrase that contrasts this style with the style of traditional poets.

    None of the other answer choices is correct because none addresses the true subject of the sentence as a whole, which seeks to contrast the styles of traditional and slam poets.

  35. C. Their work is designed to be seen in performance rather than read.

    The concluding sentence aims at a characterization of the slammers’ work as a whole, and to say that it “is designed to be seen in performance rather than read” sums it up nicely.

    The right answer is not Choice (A) because this vague and obvious statement adds very little to the paragraph. It is true, but it could just as well not be there, which isn’t what you want from a conclusion.

    The right answer is not Choice (B) because, though true, it’s a minor, unnecessary detail. Another answer choice performs a much more logical function in this context.

    The right answer is not Choice (D) because speculation about slammers using props in the future is an odd tangent and not the most effective way to end the paragraph and transition into the next.

    The right answer is not Choice (E) because changing the subject to slammers making money for their venues is a tangent that is never addressed elsewhere.

  36. A. Ironically

    The fact that each group thinks it’s being oppressed by the other is ironic, so “Ironically” is a perfectly logical way to begin the sentence.

    The right answer is not Choice (B) because the sentence is not a result of what was being discussed immediately prior, so “Therefore” doesn’t make sense as a transition word.

    The right answer is not Choice (C) because the emphatic “Undeniably” adds little in this context. This portion of the passage is not weighing evidence and/or attempting to persuade the reader.

    The right answer is not Choice (D) because the sentence is not a counterpoint to the previous one, so “on the other hand” doesn’t make sense as a transition.

    The right answer is not Choice (E) because you don’t need to say “In conclusion” just because it’s the last sentence. It would seem out of place here.

  37. B. a magazine article

    You can tell this citation is for a magazine article because it includes a month in the publication date, it lists two titles (the article in quotation marks, followed by the title of the publication itself in italics), and it notes an issue number and the page numbers of the article.

    The right answer is not Choice (A) because the inclusion of a month in the publication date, as well as the two titles, is a clue that the cited work is not a book.

    The right answer is not Choice (C) because newspapers come out daily, not monthly, so a magazine is more likely.

    The right answer is not Choice (D) because blogs don’t have page numbers.

    The right answer is not Choice (E) because the lack of a specific date (day instead of just month), the presence of two titles, and the inclusion of page numbers are all clues that the cited work is not a speech.

  38. C. a primary source represents original research, whereas a secondary source contains information already compiled by someone else.

    These are the correct definitions of “primary source” and “secondary source.”

    None of the other choices is the correct definition of primary source and secondary source.

  39. B. epigraph.

    A quotation below the title and above the main body of the text is called an epigraph.

    The right answer is not Choice (A) because an epigram is any saying or proverb.

    The right answer is not Choice (C) because epistle is a fancy word for a letter (the kind you mail).

    The right answer is not Choice (D) because epithelium is a type of animal tissue in biology.

    The right answer is not Choice (E) because epitaxy refers to the deposition of crystal in geology.

  40. B. a Works Cited page lists only cited sources, whereas a Bibliography lists every source consulted.

    These are the correct definitions of Bibliography and Works Cited page.

    None of the other answers give the correct definitions of Bibliography and Works Cited page.

Argumentative Essay

Take a look at the following essay written in response to the opinion that’s presented in Chapter 16. To score your own essay, flip back to Chapter 11, where you can find a checklist to help you evaluate your own writing.

  • The idea that “high-school and college creative-writing classes should seek to foster an emotionally nurturing environment above emphasizing the development of technical skill” is the kind of recommendation that almost anyone would want to support when he or she first hears it. It pits kindness and sensitivity against elitism, and who would want to appear to be on the side of the elitists? The problem with it, however, is the same as the problem with many statements: It oversimplifies a complex issue into a simple either/or problem, and it fails to consider that not everyone who works with young people has the same job to do.
  • Would most young people who sign up for creative-writing classes prefer to vent about personal problems than to be nitpicked about whether their work is good enough for publication? Of course they would. But young people would probably rather vent about personal problems in math class instead of doing math too — that’s just what young people are like. Yes, creative work certainly has more to do with personal expression than math does, but a class in school is still a class in school, and the students are in the class because there’s something they’re supposed to be learning. It may make a teacher feel like a great guy to tell everybody that his or her poem or story is fantastic just the way it is and then let the class use discussion time simply to share their feelings, but nobody learns anything that way.
  • A lot of young people struggle with issues in their lives, and it would help them to talk about those issues in a supportive environment. But that’s what a therapist is for. Therapy is great, and many kids could benefit from it, but that doesn’t mean it’s the teacher’s job. The writing teacher isn’t obligated to be everyone’s therapist instead of teaching writing, any more than it’s the job of the math teacher or the science teacher to be a therapist instead of teaching math or science. If the writing teacher would rather help kids work out their problems than teach them how to write, then she should get a degree in psychology and become a therapist. And more importantly, a kid with a serious problem should be seeking help from someone who is specifically trained to address it instead of assuming that the English teacher is qualified to do so.
  • Perhaps most troubling is the fact that the question lumps together “high-school and college creative-writing classes” as though they are the same thing. A ninth-grader and a college student aren’t taught the same things in the same way in any other subject, so why should they be taught writing in the same way? There’s a world of difference between a 15-year-old and a 20-year-old, and what seems sensitive and encouraging to the former might seem patronizing and pointless to the latter. High-school freshmen may sign up for a creative-writing elective because it seems like an easy class, but a young adult who registers for a college course in the same subject may be considering a career as a writer and looking for tough and honest feedback. If the teacher just smiles and praises this student instead of helping him hone his skills as much as possible, then the college is essentially stealing his money, because he isn’t learning what he paid to learn.
  • This isn’t to say that writing teachers should be insensitive. Nobody is a brilliant writer right off the bat, and it often takes years of encouragement about work that isn’t truly very good before a young person manages to pen anything worthwhile. But it’s still the students’ work that should be discussed in the classroom, rather than their personal lives.

Source-Based Essay

Take a look at the following essay written about the paradox of wanting to see things as they are versus appreciating them in a way that makes emotional sense. To score your own essay, flip back to Chapter 11 and use the checklist to help you evaluate your own writing.

  • To a great extent, the history of human civilization has been a battle between those who seek to comprehend the natural world as it really is and those who are more concerned with the feelings, experiences, and behavior of the human beings who live in it. This conflict could be said to have reached its apex in the 20th century, during which science and technology advanced by breathtaking leaps and bounds even as human suffering and existential despair were exasperated by countless wars of unprecedented scale. Now, in the early years of the 21st century, we understand the world more clearly than ever before from a scientific and philosophical perspective, but we also feel more lost in that world than we ever have.
  • According to Jeremy Wisnewski, the philosophical discipline of phenomenology, as developed by such thinkers as Edmund Husserl and Martin Heidegger, is an attempt to “set aside our preconceptions and to uncover the actual being of things as they reveal themselves to us” and “to see past our preconceptions into the heart of things” (Wisnewski, 2009). No thinking person could argue that this is not, in theory, an admirable goal. Many of the people we love would not be here today had science not succeeded in doing this to a significant extent. No one with a friend or relative whose life was saved by modern medical science could wish for a return to the days when human beings interpreted even everyday events like illness in personalized terms of spells and demons instead of in a coldly objective — and therefore effective — manner.
  • On the other hand, however, people want to enjoy their lives, not merely to stay alive for the sake of doing so. We evolved our capacities for reason and intellect so we could bond and live in harmony with one another, not because we had some pressing need to figure out what is going on in the middle of a black hole. There is a wide range of subjective human experiences of existence, and what is fascinating to a philosopher or a physicist might well be frightening to the average person. As George Tschner pointed out, “mythology is a figurative and metaphorical way the human intellect grasps its world and answers and resolves some of the most fundamental questions” (Tschner, 2009). The tales of Odysseus’s decade-long attempt to return home or of Orpheus’s descent into the underworld in search of his lost bride will never help us cure a disease or colonize an alien planet, but they were never intended to do so. A viewpoint that constitutes a wrong answer to one question may be the right answer to another.
  • To return to Wisnewski’s words, yes, we humans do “impose particular theories onto phenomena and insist that they conform to our preconceived notions about how the world is” (Wisnewski, 2009). But this is not a moral or an intellectual failing so much as it is something we evolved to do for a reason — and that reason is that it keeps our awareness of the cold, vast, unfeeling “reality” of existence from driving us to despair and madness. The scientific and philosophical methods are not a model for how everyone should live, and science and philosophy admit as much by reminding us that their disciplines are concerned with reality and results rather than with ethics and emotions. Objectivity and reason are not wrong, of course, and more spiritually inclined people would do well to stop fearing and demonizing them, but just because they are not wrong, that doesn’t mean that they are how everyone should think all the time. The scientific method is a means to certain ends, and the mythological method is a means to certain other ends. Both missions are admirable and necessary, and both ways of seeing the world are therefore respectable.

Part 3: Mathematics

  1. A. decrease of 6

    Since the value of w in the equation images is increased by 10, the value of the left side of the equation is increased by 30. That is because the coefficient of w is 3. For every 1 the value of w is increased, the value of 3w is increased by 3 since 3w is 3 times greater than w. In order for the left side to maintain its value, the increase of 30 must be accompanied by a decrease of 30. That means the value of 5u must decrease by 30. If u is decreased by 6, 5u is decreased by 30, and the value of images does not change, so the new pair also makes the equation true.

  2. E. 76
    images

    By adding the four side measures in terms of x, you get the expression images for the perimeter. If you set that equal to 46 and solve the equation, you can determine that images. Then, you can put 2 in for x in the length and width expressions to find that the length is 19 and the width is 4. Because the area of a rectangle is length times width, the area of this rectangle is images, which is 76.

  3. C. images

    Magnitude is the same as absolute value, or positive distance from 0. You can get the absolute values of all the numbers by dropping the negative signs from the negative numbers and leaving the positive numbers positive. If you convert all the numbers to the same form, such as decimals or improper fractions, you can more easily see the order of the magnitudes from least to greatest.

  4. A. 93

    Because a mean is the sum of all items of data divided by the number of items of data, you can add all the test scores and a variable representing the missing test score and then divide that sum by the number of test scores, which is 6. If you set that ratio equal to 85 and solve for the variable, you can determine that the missing test score is 93.

    images
  5. B. Divide both sides of the equation by 5.

    Brady subtracted 7 from both sides of the original equation, so the resulting equation is images. The goal in solving an equation is to get the variable by itself on one side so that the other side can represent the value of the variable. That can be achieved here by dividing both sides by 5. The resulting equation would be images, which is a statement of the solution to the original equation.

  6. D. 18.00 ounces

    If the recipe calls for images cup of tomato sauce per pizza and 9 pizzas are desired, multiplication is required. To find how many ounces this is requires use of the conversion factor provided:images. Putting all of this information into one:

    images
  7. A. 20%

    To calculate percentages, first get the total number of people surveyed. images. images. Therefore, images.

  8. C. None of the marketing experts are computer technology experts.

    In the Venn diagram, the figures representing marketing and computer technology do not overlap. That means none of the marketing experts are also computer experts. None of the other statements expresses a correct situation concerning overlapping areas of expertise.

  9. A. images

    By using reverse FOIL for the top and bottom expressions, you can see that the top is images and the bottom expression is images. Because images is a factor of both the top and bottom expressions, it can be cancelled, leaving images.

  10. D. 65

    65 is 1 greater than 64, which is the square of the integer 8. 65 is not a prime number. All of the other choices are 1 greater than the square of an integer, but they are prime numbers.

  11. A. quadrilateral, B. triangle, and C. trapezoid

    The answer to this question comes down to what figures can result from dividing a triangle into two parts. Although planes are two-dimensional, they have no depth. When they intersect triangles, they can only cut them in the ways that lines coplanar to the triangle would. Cutting a triangle into two parts with linear intersections at their vertices can only create smaller triangles, and cutting triangles just through the sides will only separate them into quadrilaterals and triangles. Because the sides of a triangle are never parallel, it would be impossible to divide a triangle into two parts with one being a square or any other type of parallelogram, and a triangle does not have enough sides for a pentagon or any other type of polygon with more than four sides to be formed. However, a linear separation that is parallel to a side of a triangle can mark off a trapezoid.

    image
  12. E. images

    You can determine the equation from the table alone by getting the slope and y-intercept. The slope is the amount y changes each time x increases by 1, and the y-intercept is the value of y when x is 0. With those values, you can determine an equation using the slope-intercept formula, which is images, for which m is the slope and b is the y-intercept. The slope-intercept form of this equation is images. That equation can be converted to the form images. However, there is a quicker way to answer this question. You can put any x and y pair in the table into each equation until you find an equation that works for the ordered pair, and then you can try the other pairs to make sure they make the equation true. Every ordered pair listed in the table works as a solution to images. In other words, for each ordered pair in the table, putting the x-value in for x in the equation and the y-value in for y in the equation makes the equation true.

  13. C. 260 cm2

    This problem requires you to take what you know and use it to work your way to what you don’t know.

    The formula for the volume of a pyramid is images. The volume and height of the pyramid are given, so you can put those values into the formula and solve for B, base area. This process reveals that the base area is 144 cm2.

    The base is a square, so the sides have equal measures. The measure of a side times itself gives the area of the square, so a side is equal to the square root of the area. The square root of 144 is 12, so each side is 12 cm.

    That means the distance from the center of the square base to the midpoint of one of its sides is 6 cm. The center-midpoint segment, the segment representing the height of the pyramid, and a segment representing the slant height of the pyramid form a right triangle. By using the Pythagorean theorem, images, you can determine that the slant height of the pyramid is approximately 10.8 cm.

    With the information gathered so far, you have what you need to use the formula for the lateral area of a pyramid, which is images times perimeter times slant height. By putting the known values into the formula, you can conclude that the lateral area of the pyramid, to the nearest whole number, is 260 cm2.

  14. C. This week’s sales are higher by 9 cars.

    Based on the graph, the following numbers of cars were sold: Monday — 3; Tuesday — 5; Wednesday — 6; Thursday — 4; Friday — 9. That brings the total for the week to 27 cars sold. If the weekly average for last year was 18, then you can subtract to compare. images. The verbiage of that is that this week’s sales are higher than last year’s average by 9 cars, Choice (C).

  15. A. The mean is greater than the median.

    The mean can be determined by adding all the data figures and then dividing by the number of data figures. The mean is 63.666…

    The median is found by putting the data in order from least to greatest and determining the middle number or the average of two middle numbers. In this case, it is a single middle number, 76.

    The mode is the number that appears the highest number of times in the set. That number is 41.

    Using the exact numbers for mean, median, and mode, you can compare them and determine that the only false statement is Choice (A). The mean is 63.666…, which is not greater than the median, 76. All of the other statements are true.

  16. E. The domain element –1 is paired with two range elements.

    A relation is a function as long as every domain element is paired with only one range element. The relation here has the domain element –1 paired with both 3 and 9. That’s why the relation isn’t a function. None of the other choices concern a domain element being paired with more than one range element.

  17. B. 26.25

    There are two similar triangles in this figure. Corresponding sides in similar triangles are proportional.

    Therefore, a proportion can be set up to solve for y.

    images

    Then cross-multiply.

    images
  18. C. 41 miles

    Identify what the question is asking for first: the total number of miles driven. What else did she do on her 1.5-hour or 90-minute trip? She was at the gym for 45 minutes. That leaves 45 minutes of drive time or 0.75 hours.

    images
  19. 44

    In order to find the range, it is necessary to find the lower limit, 21, and the upper limit, 65. Then subtract these two: images.

  20. D. 36

    The question is essentially asking this: 27 is 75 percent of what number? To find the answer to the question, you can translate it into mathematical language and then solve for the unknown. You can use the equation images, where x represents the unknown. Solving the equation reveals that x is 36. The other choices can result from incorrect translations of the question.

  21. D. (–7, 4), (–9, 10), (2, 3), and (2, 12)

    The vertices of the quadrilateral before the translation are (–3, –3), (–5, 3), (6, –4), and (6, 5). To translate the quadrilateral 7 units up and 4 units left, add 7 to each y-coordinate and subtract 4 from each x-coordinate. The resulting coordinates are (–7, 4), (–9, 10), (2, 3), and (2, 12). The other choices can result from performing the wrong operations on the original coordinates or placing them in incorrect orders, or both.

  22. B. 18.8%

    To find percent increase, do the following:

    • images
    • images
  23. A. 3

    To begin, substitute 3 for x in the equation. Then work to get y on one side by itself.

    images
  24. C. 10

    The sum of the Japanese and Greek percentages is 22 percent. That figure subtracted from the percentage for Mexican (24 percent) is 2 percent. Two percent of 500 is 0.02(500), which is 10. You could find the numbers of people represented by each percentage first, but finding the difference of the percentages first is probably easier.

  25. 8

    By placing each variable value into the equation in place of the variable, you get an equation without variables. From there, you can follow the order of operations. A fraction bar is a grouping symbol, so the parentheses on top are within a grouped part of the fraction. The value within the parentheses is 2, the square of which is 4. The product of 4 and 4 is 16, and images, or images, is 64. Therefore, the value of the numerator is 64. The denominator is images, so the value of the denominator is 8. A fraction represents the numerator divided by the denominator: images.

    images
  26. B. asking questions of every tenth worker who walks through the main entrance for work

    Asking questions of every tenth person who walks through the main entrance would most likely result in answers from people of varying departments and other categories that may affect poll results. The other choices involve samples that lack diversity and could therefore cause biased results.

  27. D. images cm3

    The formula for the volume of a cone is images, where B represents base area and h represents the height of the cone. The base of a cone is a circle, so the base area of a cone is images, with r representing the radius of the base. Using the formula, you can determine that the volume of the bigger cone is images cm3, and the volume of the smaller cone is images cm3. The volume of the larger cone minus the volume of the smaller cone is therefore images cm3.

    images
  28. B. To determine which graph represents the scenario, you must understand the x and y axes. It is important to watch what the distance is doing as time passes. Based on the information given, there should be an increase in distance with a slope that moves up diagonally, then a point where the distance does not change, and then a time where distance continues going up again just like the initial diagonal slope. The only one that gives us this is the graph in Choice (B). For Choice (A) to be correct, Bobby would have to be going in the opposite direction or retracing his steps. Choice (C) indicates that as time passed, Bobby stayed in the same spot. Choice (D) shows an increase in direction at a constant rate, staying in one spot for a period of time, and then going back to his original spot at the same rate.
  29. C. 36

    You can use a variable to represent Eric’s age and 9 times that variable to represent Natasha’s age. You can call Eric’s age x and Natasha’s age 9x. In 10 years, their ages will be images and images. Then you can write an equation based on the situation described. The sum of their ages in 10 years can be represented by images, which is equal to 60. By solving the equation images, you can determine that x is 4, so Natasha’s age is 9(4), or 36. Choice (A) is the sum of their ages in 10 years, Choice (B) is Eric’s age in 10 years, and Choice (D) is Eric’s current age. Choice (C) results from subtracting Eric’s current age plus 10 from 60.

    images
  30. D. 1/98

    The hat contains 14 marbles, 2 of which are blue and 1 of which is orange. Because probability is determined by dividing the number of qualifying outcomes by the number of possible outcomes, the probability of picking a blue marble is 2/14, or 1/7. The probability of picking an orange marble is 1/14. The probability of picking both is the product of the two fractions, or 1/98.

  31. D. 11

    First, you must set up an equation, using variables to represent the currencies. If q represents quarters and d represents dimes, you can set up the following equation:

    images

    Then to get only one variable in an equation, determine the relationship between d and q. The question tells you he has two more quarters than dimes. Therefore:

    images

    Now substitute this new expression in for q:

    images

    So there are 9 dimes. Because there are two more quarters than dimes, there are 11 quarters.

  32. B. 7 m and C. 9 m

    When two segment measures are given, there is a range of possibilities for what the measure of a third segment can be to make forming a triangle possible. That range is between, and not including, the positive difference to the sum of the two given measures. That is because no two segments can join at their endpoints and form an angle and still join their other endpoints to the endpoints of a segment that is longer than both of them combined. Any combination of two segments must cover more distance than the other segment. In this case, the two given segment measures are 8 m and 3 m. Their sum is 11 m, and their positive difference is 5 m. Therefore, any third segment that could join endpoints with the 8 m and 3 m segment must be between 5 m and 11 m. They cannot have those measures. The measures they have must be between 5 m and 11 m. The choices that are between those measures are Choices (B) and (C).

  33. images

    The probability of an event is the ratio of the number of qualifying outcomes to the number of possible outcomes. For a single quarter toss, landing on tails is 1 qualifying outcome out of 2 possible outcomes. Thus, the probability that a quarter toss will result in landing on tails is images. That is the case no matter how many times the quarter has already landed on heads or tails. The probability remains the same.

  34. A. images

    images, –3.71, and all the answer choices can be written as improper fractions, mixed numbers, or decimals. When they’re all in the same form, you can see which numbers are greater than others. For example, images is the only choice that is between images and –3.71, which can be written as images and images by converting to mixed numbers.

  35. D. 327.5

    The set of data doesn’t have just one middle number; it has two middle numbers. When the numbers are in order from least to greatest, the two middle numbers are 218 and 437. The mean of those two numbers is the median of the set of data, and that mean is 327.5, so 327.5 is the median of the set of data.

  36. A. images

    Replace x with 6 in the rational expression.

    images

    By using the order of operations and simplifying the resulting fraction, you can conclude that if x = 6, the expression = images. The other choices result from using incorrect orders of operations.

  37. C. 10

    Because the rectangles are similar, their lengths and widths are in proportion. You can set up a proportion involving ratios of corresponding sides. You have several options for this. One proportion you can use is images. The solution to the proportion is the value of q, which is 10.

    images
  38. B. –57

    By solving the first equation, you can find that the value of k is 5. Then you can put 5 in for k in images to get –57.

    images
  39. C. 6

    x is 20 less than y, or images.

    z is 14 less than y, or images.

    The issue is how images compares to images. You can subtract one from the other to find out.

    images, or images, which is 6.

    Because images is 6, z is 6 greater than x.

  40. D. images

    Every time a coin with heads and tails is flipped, the probability that it will land on heads is images. The probability of multiple events is the product of the probabilities of the individual events. images.

  41. A. 20 percent

    Mark hit the ball 48 out of 60 times he was up at bat, which converts to 80 percent. If Mark hit the ball 80 percent of the time, then he struck out 20 percent of the time he was up at bat.

  42. D. Tim is 54, and Sarah is 27.

    To set this up in mathematical language, start with what you know first. Tim is twice Sarah’s age, so: images.

    Next, Rachel is one-third Tim’s age, so: images.

    Then substitute one term for the other:

    images

    If Sarah is 27, then Tim is 54 because he is twice Sarah’s age. Choice D is the correct answer.

  43. D. 94

    The angle that is w degrees is a vertical angle to an angle in the quadrilateral, as is the 110-degree angle. All vertical angles are congruent, so those are the measures of two of the angles of the quadrilateral. The 117-degree angle is supplementary to another angle in the quadrilateral, and the 93-degree angle is vertical to the remaining one. The interior angles of a quadrilateral have a sum of 360 degrees. By adding the angle measures of the quadrilateral and setting the sum equal to 360 degrees, you can solve for w. The result is 94 degrees. The other choices result from combinations of mixing the rules of supplementary angles with those of vertical angles.

  44. C. 6.

    To read a dot plot, understand what the dots represent. Here each number represents the number of books, and the dots tell how many students read that particular number of books. So, if you are looking for how many students read one book, you go to the number 1 and count the dots above it. There are 6 dots, so six students read one book. Be careful. Don’t let the fact that there is only one dot in other places trip you up. Those dots indicate that only one person read 3, 5, 7, 8, or 9 books.

  45. B. images

    A third of r can be written as images and p less than that as images. All of the other choices involve false representations of those expressions.

  46. E. 21%

    This question involves the reading of a graph. You can note significant points on the graph such as the following: In 2010, the percentage was about 6; in 2012, about 9; in 2014, about 12; and in 2016, about 15. That shows an increase of 3% every two years. From 2016 to 2020 is four years, so there should be an increase of 6% from the last data point. images. Therefore, if this trend continues, in 2020, about 21% of Americans should be using online dating sites.

  47. C. images

    What has been described looks like the image that follows. You should try to sketch this out on test day so that you can get a visual of what is being represented.

    image

    You can see here that the radius of the circle (needed to find the area) is the same as half the length of a side of the square. If the side of the square is 14, then the radius would be 7. The formula for area of a circle is: images.

    images
  48. B. (4, 7)

    x is 4, and y is 7. One method you can use to find the solution is elimination. By multiplying both sides of the top equation by 2 and both sides of the bottom equation by –3, you get two equations in which the coefficient of x is the same.

    images

    You can add the resulting bottom equations to get rid of x.

    images

    Then solve for y, which is 7.

    images

    You can then put 7 in for y to determine that x is 4.

    images

    You can put the solution into both equations to see that it works for them.

  49. C. 19

    To find the number of participants in the tournament based on this stem-and-leaf plot, simply count the number of leaves. There are 19 numbers under the “leaf” side. So there are 19 males who signed up for the tournament. The stem side should only be used to indicate their full ages: 10, 13, 16, 21, and so on.

  50. D. 159

    To solve this function, all you have to do is substitute images anywhere there is an x:

    images
  51. E. images

    To solve an inequality, follow the rules of solving equations. However, the inequality sign must change directions if you switch the sides or multiply or divide both sides by a negative number.

    images
  52. C. 6

    The line graph increases at a constant rate, and it indicates that the number of millimeters the plant is tall is very close to half the number of days the plant has been growing. Therefore, at 12 days of growth, the plant will be approximately 6 mm tall. It will be much closer to that measure than any of the other choices.

  53. A. Move the decimal three places to the left and make the exponent three higher.

    Scientific notation consists of a number in which a single digit is followed by a decimal point (except in some cases of single-digit whole numbers, for which decimal points are not required) and the number is multiplied by 10 with an exponent. The number in the question has four digits before the decimal, so the decimal must be moved three places to the left. To keep the original value, the exponent of 10 has to increase by three to make up for moving the decimal three places to the left.

    images
  54. A. a positive slope line

    A line of best fit is a single line that represents the nature of the points in a scatterplot. If the points in the chart were plotted on a coordinate plane, they would give a line similar to the one that follows:

    image

    This indicates a positive slope; whereas one entity is increasing, so is the other. Based on this particular scenario, as the heights of the players increases, so do their weights. Be careful that you don’t read too much into the data. Base conclusions on what is presented. Even if you know of a case where the heights and weights don’t necessarily correlate, you can’t bring that information to this problem.

  55. C. E. images

    The way to multiply unit rates in order to get a unit rate with the desired units is to multiply unit rates that are equal to 1 because the numerator and denominator are equal but expressed with different units, such as images. A desired unit should be in just a numerator if you want it in the numerator of the product and in just a denominator if you want it in the denominator of the product. Any unit you want to cancel needs to be in a numerator and a denominator, once for each, or at least an equal number of times for each. Any value that is a factor of a numerator and a denominator, an equal number of times for each in a product of fractions, can be canceled. For this problem, images needs to be multiplied by a unit rate that has a number of inches in the numerator and a number of feet in the denominator. The unit rate that fits that standard and is equal to 1, and therefore does not cause a change of value when a value is multiplied by it, is images. Multiplying by that unit rate will result in a product that has inches in the numerator. Seconds can already be in the denominator as a result of cancellation.

    images
  56. B. 30 m2

    The width of the rectangle, 3 meters, is also the measure of a leg of each triangle. Because the triangles are congruent, each one has a hypotenuse of 5 meters. With those measures, you can use the Pythagorean theorem to determine that the other leg of each triangle has a measure of 4 meters. Using the formulas for rectangle area (images) and triangle area images, you can determine that the area of the rectangle is 18 m2 and the area of each triangle is 6 m2. The sum of the areas of the triangles and the area of the rectangle is the area of the overall figure, and that area is 30 m2.

    • images
    • images
    • images

Answer Key

Part 1: Reading

  1. E
  2. C
  3. C
  4. D
  5. A
  6. B
  7. A
  8. A
  9. E
  10. C
  11. B
  12. B
  13. C
  14. D
  15. E
  16. C
  17. C
  18. C
  19. A
  20. B
  21. A
  22. E
  23. C
  24. B
  25. C
  26. E
  27. D
  28. A
  29. D
  30. C
  31. B
  32. D
  33. D
  34. A
  35. C
  36. E
  37. B
  38. D
  39. C
  40. C
  41. B
  42. B
  43. A
  44. E
  45. C
  46. D
  47. 2010
  48. C, D
  49. B
  50. A
  51. E
  52. C
  53. B
  54. D
  55. A
  56. C

Part 2: Writing

  1. E
  2. D
  3. A
  4. B
  5. C
  6. D
  7. E
  8. B
  9. C
  10. E
  11. A
  12. B
  13. B
  14. D
  15. A
  16. D
  17. D
  18. B
  19. A
  20. D
  21. A
  22. C
  23. E
  24. C
  25. D
  26. E
  27. B
  28. C
  29. A
  30. B
  31. B
  32. A
  33. D
  34. E
  35. C
  36. A
  37. B
  38. C
  39. B
  40. B

Part 3: Mathematics

  1. A
  2. E
  3. C
  4. A
  5. B
  6. D
  7. A
  8. C
  9. A
  10. D
  11. A, B, C
  12. E
  13. C
  14. C
  15. A
  16. E
  17. B
  18. C
  19. 44
  20. D
  21. D
  22. B
  23. A
  24. C
  25. 8
  26. B
  27. D
  28. B
  29. C
  30. D
  31. D
  32. B, D
  33. images
  34. A
  35. D
  36. A
  37. C
  38. B
  39. C
  40. D
  41. A
  42. D
  43. D
  44. C
  45. B
  46. E
  47. C
  48. B
  49. C
  50. D
  51. E
  52. C
  53. A
  54. A
  55. C, E
  56. B
..................Content has been hidden....................

You can't read the all page of ebook, please click here login for view all page.
Reset
18.119.131.72